Tải bản đầy đủ (.docx) (20 trang)

Phần 2 ok

Bạn đang xem bản rút gọn của tài liệu. Xem và tải ngay bản đầy đủ của tài liệu tại đây (230.27 KB, 20 trang )

1. XÁC ĐỊNH SỐ HẠNG TỔNG QUÁT
1.1. DỰ ĐOÁN SỐ HẠNG TỔNG QUÁT VÀ CHỨNG MINH BẰNG QUY NẠP
u1 11

un 
u 10un  1  9n, n  N

Bài 1. Cho dãy số
xác định bởi :  n 1
. Xác định số hạng tổng quát của
dãy đã cho.
Hướng dẫn giải
Ta có:
u1 11 10  1
u2 10.11  1  9 102 100  2
u3 10.102  1  9.2 1003 1000  3

u 10 n  n  1
Dự đoán: n
Chứng minh theo quy nạp ta có
u1 11 101  1 , công thức  1 đúng với n 1 . Giả sử công thức  1 đúng với n k ta có uk 10k  k .
u
10  10k  k   1  9k 10k 1   k  1
Ta có: k  1
 1 đúng với n k  1
Công thức
n
Vậy un 10  n , n  N .
Bài 2. Cho dãy số (un ) biết

u1  2



un 3un  1  1, n 2

. Xác định số hạng tổng quát của dãy.

Hướng dẫn giải
1
3
1
1
un 3un  1  1  un  3un  1   un  3(un  1  )(1)
2
2
2
2
1
1 5
vn un   v1 u1  
2
2 2
Đặt
(1)  vn 3vn  1 , n 2
(v ) là cấp số nhân với công bội là q 3 .
Dãy n
 5 n 1
.3
2
Nên
.
1  5 n 1 1

un vn   3  , n 1, 2,...
2 2
2
Do đó
vn v1.q n  1 

1.2. DÃY SỐ CHO BỞI CÔNG THỨC TRUY HỒI
1.3. PHƯƠNG TRÌNH ĐẶC TRƯNG
1.4. PHƯƠNG PHÁP DÃY SỐ PHỤ
1.5. DÃY SỐ SINH BỞI PHƯƠNG PHƯƠNG TRÌNH
1.6. SỬ DỤNG PHÉP THẾ LƯỢNG GIÁC
1.7. CÁC DẠNG KHÁC
2. MỘT SỐ DẠNG TỐN LIÊN QUAN ĐẾN TÍNH CHẤT CỦA DÃY SỐ

Bài 3. Cho dãy

 xn 

xác định bởi:

Chứng minh rằng

∀ n∈N

{x1=1;x2=3 ¿ ¿¿¿
2
ta có 2 xn  2 là số chính phương.


Hướng dẫn giải:

Dễ thấy xn  Z n  N , n 1
Ta có xn = 6 xn  1  xn  2  xn  3 xn  1 3 xn  1  xn  2
2




từ đó ta có

2

x n−6 xn . x n−1 +9 x n−1 = 9 x 2n−1 −6 x n−1 . x n−2 +x 2n−2
2
2
x n−6 xn . x n−1 + x n−1 = x 2n−1 −6 x n−1 . xn−2 +x 2n−2

x 2n−6 xn . x n−1 + x2n−1 =



2

2

2

x 2−6 x 2 . x1 + x 1=−8

2


x n−1 −6 x n . x n−1 +x n +8=0

(1)

'
Vì nên phương trình (1) phải có nghiệm nguyên . Do đó (1) có Δ phải là số chính phương

Tức là tồn tại sao cho

'

Δ=

Từ (2) ta suy ra k phải là số chẵn


Vậy

2

2

2

⇒ k=2 m ;m∈N

9 x 2n−( x 2n +8 )=8( x 2n−1 )=k 2

2 x 2n −2


2

9 x n−( x n +8 )=8( x n−1 )=k

kN

 an  n

(2)

 0

là số chính phương.

Bài 4. Cho dãy với được xác định bởi:

a) Chứng minh an chia hết cho n với mọi giá trị nguyên dương của n .
 a1 1; a2 2; a3 6; a4 12
an

bn 
n . Chứng minh tồn tại vô số  an 4 2an 3  an 2  2an 1  an , n 1 số nguyên dương
b) Đặt
n để 2015 là một ước của bn .
Hướng dẫn giải
a) Ta có b1 1; b2 1; b3  2; b4 3
Dễ thấy bn  Fn với n 1; 2;3; 4 .

b 
F 

Bằng quy nạp ta chứng minh dãy n trùng với dãy n
Thật vậy:
Mệnh đề đúng với n 1; 2;3; 4 Giả sử mệnh đề đúng đến n  3 . Khi đó ta có:
(n  4)bn 4 2(n  3) Fn 3  ( n  2) Fn 2  2(n  1) Fn 1  nFn
 n  4  Fn4
Dùng công thức của dãy Fibonaci : Fm2  Fm1  Fm ta dễ dàng biến đổi vế phải thành
suy ra bn 4  Fn 4 .
Vậy mệnh đề đúng với n  4 , do đó nó đúng với mọi n nguyên dương.
Điều đó chứng tỏ an luôn chia hết cho n với mọi n nguyên dương.

r 
b) Gọi rn là số dư của bn cho 2015 với n 1; 2;3; 4 Trước tiên ta chứng minh n là một dãy
tuần hồn. Thật vậy:
Ta có : bn 2 b n 1 bn  rn 2 rn 1  rn (mod 2015)


Vì có vơ hạn các cặp ( r1; r2 ), ( r2 ; r3 ),...(rn ; rn 1 ) nhưng chỉ nhận hữu hạn giá trị khác nhau nên tồn tại ít nhất
hai phần tử của dãy trùng nhau. Ta giả sử là ( rm ; rm1 ) (rmT ; rmT 1 ) (với T là một số nguyên dương).
Ta chứng minh (rn) tuần hoàn với chu kỳ T .
) Ta có : rm 2 rm 1  rm (mod 2015); rm T  2 rm T 1  rm T (mod 2015)
 rm 2 rmT  2 (mod 2015)
 rm 2 rmT  2
Tiếp tục như vậy ta chứng minh được: rmk  rmT k với mọi k 0 (1)
) Ta có : rm 1 rm1  rm (mod 2015); rmT  1 rmT 1  rmT (mod 2015)
 rm 1 rmT  1 (mod 2015)
 rm 1 rmT  1
Bằng quy nạp ta chứng minh được: rm k  rmT  k với k 1; 2;...; m  1 (2)
r 
Từ (1) và (2) suy ra n n 0 là một dãy tuần hoàn.
b 

r 
Bổ sung vào dãy n phần tử bo 0 thỏa mãn b0  b1 b2 suy ra r0  0 . Khi đó dãy n là dãy tuần
r 
hoàn bắt đầu từ phần tử đầu tiên r0 = 0. Do đó tồn tại vô số phần tử trong dãy n bằng 0 .Như vậy câu b
được chứng minh xong.
3. GIỚI HẠN CỦA DÃY SỐ
3.1. TÍNH GIỚI HẠN BẰNG ĐỊNH NGHĨA

Bài 5.

Cho dãy số

 an 

1

 a1 a  a


3
2
 an 1  2an  2an  2

3an 2  4an  1
xác định bởi : 

a 
a 
Chứng minh rằng với mọi số thực a 0 thì dãy n hội tụ. Tùy theo a , hãy tìm giới hạn của dãy n .
Hướng dẫn giải

1
a  2
a
Nếu a  0 thì
(do bất đẳng thức AM-GM)
Nếu a  0 thì

 a

1
1
2
a   2
a
a
(do bất đẳng thức AM-GM) nên

*
Nếu a 1 thì a1 2 . Ta chứng minh: an 2, n  
Hiển nhiên a1 2 .

2.23  2.22  2
ak 2  ak 1 
2
3.2 2  4.2  1
Giả sử
lim an lim 2 2
Vậy
.
a  0


*
. Nếu  a 1 thì a1  2 . Ta chứng minh an  2 n  
Rõ ràng a1  2 .
Giả sử ak  2 . Ta chứng minh ak 1  2


2ak 3  2ak 2  2
2
 2  2 ak  a k  2   0
2
3ak  4ak  1
( đúng)
a 
Ta chứng minh n là dãy giảm, thật vậy :
2
 a 3  2a 2  an  2   an  1  an  2 
n, an 1  an  n 2 n

0
3an  4an  1
3an 2  4an  1
( do tử âm, mẫu dương vì

2 7
 an 
3
3an 2  4an  1  0  

2 7

 an 
3

2 7
an  2 
 3an 2  4an  1  0
3

)
ak 1  2 

 an 

a 
giảm và bị chặn dưới  n có giới hạn là L .
2a 3  2an 2  2
2 L3  2 L2  2
lim an 1 lim n 2

3an  4an  1
3L2  4 L  1

 L 2  an  2  L  1 
Vậy lim an 2 .
. Nếu a  0 thì a1  2 . Tương tự, ta có:
2
 an 3  2an 2  an  2   an  1  an  2 
n, an 1  an 

0

3an 2  4an  1
3an 2  4an  1

a 
tăng. Hơn nữa n bị chặn trên bởi  1 , thật vậy
2ak 3  2ak 2  2
2
ak 1   1 
  1   ak  1 (2a  3)  0
2
3ak  4ak  1

nên

 an 

a 
a 
Vậy n tăng và bị chặn trên  n có giới hạn là L .
an   1, n , an 1  an  0, n
2 L3  2 L2  2
 L  1  an   1 L 2 
3L2  4 L  1
Vậy lim an  1
L

Tóm lại: + Nếu a 1 thì lim an 2
a  0

+ Nếu  a 1 thì lim an 2

Bài 6.

+ Nếu a  0 thì lim an  1 .
x 
Cho dãy số n được xác định bởi

 x1  0

1 2 3
2015

*
 xn 1  xn  x  x 2  x 3    x 2015  n   
n
n
n
n


Tìm giới hạn của dãy nxn khi n   , với  là số thực cho trước.
Hướng dẫn giải


Dễ dàng chứng minh được xn  0, n 1 bằng qui nạp
Ta có
2


1
1

1
xn 1  xn  , n 1 xn21   xn    xn2  2  2  xn2  2 ; n 1
xn
xn 
xn

x 2  xn2 1  2  xn2 2  4    x12  2  n  1
Bởi vậy n  N, n 2 thì n
 xn  1, n 2 và lim xn 
n  

*

Với n  N , đặt

xn 1  xn 

xn  1; n 2  0  tn 

1
2 3
2015
tn  2  3  2015
 tn
xn
xn xn
xn
trong đó

t

xn2 , với t 2  3  2014  2015 (1), suy ra

2



2t
1
1
x  x  xn   tn   xn2  2  tn2  2  2 xntn  n  2
xn
xn
xn


. khi n   .

b1  x12

b xn2  xn2 1 , n 2.
b 
Áp dụng định lý trung bình Cesaro cho dãy n với  n
b  b  bn
lim 1 2
lim bn 
lim bn 2.
n  
n
ta có n  
2 suy ra n  

2
n 1

2
n

2
2
2
2
2
2
2
n 1
xn2  xn  xn  1    xn  1  xn  2    x2  x1   x1 b1  b2  bn
lim 2  .


n   x
2
n
n
n
Mà n
suy ra
n 1
lim 2 
n   x
2 như sau (chứng minh định lý trung bình Cesaro)
n

Thật vậy ta có thể chứng minh trực tiếp

 cn  : c1 x12  2; cn xn2 

xn2 1  2

với n 2,3

cn  ,  n m.
lim cn 0
*
n  
2
nên   0 tồn tại m  N sao cho
M max  ci 
Gọi
với 1 i m  1 .
 2  m  1 M 
2  m  1 M
 m  1 M  
m  
 1

m
'




m

2.
Với  ở trên tồn tại
thì
hay
n  max  m, m '
Xét
. ta có

n
n
m 1
|  i 1ci |  i m ci  i 1 | ci |  n  m  1 2  m  1 M   m  1 M  




 
   .
n
n
n
n
n
2
m
2 2
o đó theo định
n
|  i 1ci |
lim

0
n
nghĩa n  
2
2
2
2
2
2
2
n 1
xn2  xn  xn  1    xn  1  xn  2    x2  x1   x1 c1  c2  cn
lim
 .


2
n   x 2
2
n
n
n
n
. suy ra
Xét dãy

Nếu   2 thì

n.xn n.xn 2 


1
khi n  
2


 2
2
Nếu    2 thì n.xn xn .n.xn   khi n  

 2
2
Nếu    2 thì n.xn xn .n.xn  0 khi n   .


3.2. TÍNH GIỚI HẠN BẰNG CÁC CƠNG THỨC CƠ BẢN
u 
Bài 7.
Cho dãy số n được xác định như sau
u1 1

*
un 1  un  un  2   un  4   un  6   16, n  
n

Đặt

vn 
i 1

1

ui  5 , hãy tính lim vn .
Hướng dẫn giải:

*
Dễ thấy un  0, n   .
Theo bài ra ta có

un 1 

u

2
n

 6un   un2  6un  8   16 

un 1  1  un  1  un  5  

Suy ra

n

vn 
i 1

Do đó

u

2

n

2

 6un  4  un2  6un  4

1
un 1  1



1
1

un  1 u n  5

n
 1
1
1 
1
1
1
1
 


 

ui  5 i 1  ui  1 ui 1  1  u1  1 un 1  1 2 un 1  1


2
Mặt khác, từ un 1 un  6un  4 ta suy ra un 1  6un .
Kết hợp với u1 1 ta có

un  6n  1 , n  *  lim un   lim

1
un 1  1

0
.

1
1  1
lim vn lim  

2 un 1  1  2

Từ đó ta có

u 
Cho dãy số thực n

Bài 8.

*
ln  1  un2   nun 1, n  *
n



với
thỏa mãn
.

n  1  nun 
n  
un
Tìm
.
lim

Hướng dẫn giải:
*
f  x  ln  1  x 2   nx  1, x  
Với mỗi n   , đặt n
2

 x  1  n  1 0
2x
f  x 

n

1  x2
1  x2
Ta có
'
n


 x  1
f n'  x  0  
 n 1
Do đó

fn  x 

là hàm tăng thực sự trên  .


 f n  0   1  0

 1
1 

 f n  n  ln  1  n 2   0


Ta có   
f  u  0
Do đó !un   sao cho n n

lim un 0
Ta thấy n  

0  un 

1
n.


1

2 u2
n 1
lim
ln
1

u
n
 n  

 lim nun  lim  1  ln  1  un2   1

n   
Do đó:  n 
1
n ln  1  un2 
n  1  nun 


lim
 lim
 lim  nun ln  1  un2  un2  1.
n  
n  
n  
un
un



Vậy

Bài 9.

Cho dãy số

 an  , n 1

thỏa mãn

a1 1, an 

2n  3
an  1 , n 2
 b  , n 1 thỏa mãn
2n
và dãy n

n

bn  ai , n 1
i 1

. Chứng minh dãy

 bn 

có giới hạn và tìm giới hạn đó.


Hướng dẫn giải:
Ta có

2nan  2n  3 an 1  an  1 2   n  1 an  1  nan  , n  1
.
n

Do đó

bn  2  iai   i  1 ai 1  2  1   n  1 an 1 
i 1

nan 

1
, n 1
n
.

Ta chứng minh bằng quy nạp rằng
Thật vậy:
- Với n = 1, ta có a1 1 nên khẳng định đúng.

an 1 

2  n  1  3
 2n  1   1 
an 



2  n  1
 2n  2   n n  , ta cần chứng

 n 1 . Ta có
- Giả sử khẳng định đúng với n
1
 2n  1   1 

  2n  1 n  1 2n n



 2n  2   n n   n  1 n  1
minh
  4n 2  4n  1  n  1 4n3  1 3n
Bất đẳng thức cuối đúng nên khẳng định trên đúng với n  1 .
Theo ngun lí qui nạp thì khẳng định được chứng minh
1 

2 1
 2  1   n  1 an 1  bn 2
n

1


Ta có
Theo ngun lí kẹp thì dãy

Bài 10.


Cho dãy số

 an 

lim bn 2
có giới hạn và
.
4

 a1  3
n 1, n  

2
2
 n  2  a n a   n  1 a a
n
n 1
n n 1
thỏa mãn: 

 bn 


Tìm lim an .
Hướng dẫn giải

 n  2
*
Dễ thấy an 0, n   . Từ giả thiết ta có


*
Với mỗi n   , đặt

yn 

an 1

2



n2
  n  1
an

1 1

an 4 ta có y1 1 và

1
1
n2
2
2
2
2
n

2

y


n
y


n

1

n

2
y

n
y

y

y

  n1 
 
 n1
n
n 1
 n
 

2 n
4
4


 n  2
2

Do đó

2

2

2

2

4n  n  1
4
 n  1  n  2   1 
yn 
 an 
 
 ...   y1 
2 2
2
 n 1   n  1   3 
16  n 2  n  1
 n  1 n


Vậy lim an 4 .
3.3. TÍNH GIỚI HẠN BẰNG ĐỊNH LÍ KẸP
1
lim n
Bài 11. Tìm n  n !
Hướng dẫn giải
n
Trước hết ta chứng minh bất đẳng thức : n! > ( 3 ) n (*) ( n  N*).
1
Bằng phương pháp qui nạp. Thật vậy : với n =1 , ta có 1 > 3 (đúng).
k
Giả sử (*) đúng với n = k tức là : k! > ( 3 )k . Ta đi chứng minh (*) đúng với
n = k+1.
3
k
k 1
k 1
1
(1  ) k
k > ( 3 )k+1
Ta có (k+1)! = k!(k+1) >( 3 ) k (k+1) = ( 3 )k+1.
Bất đẳng thức cuối này đúng vì :
1
k k (k  1) 1
k (k  1)(k  2)....(k  k  1) 1
. 2
k
k +...+
k!

(1+ k )k =1+ k + 2!
.k =
1
1
1
1
2
k1
1
1
1
1
(1  )
(1  )(1  )...(1 
)
k +...+ k !
k
k
k < 1+1+ 2! +… + n ! <1+1+ 2 +...+ 2n 1 <
= 1+1+ 2!
1
1
1
1
1
n 1
<1+1+ 2 +...+ 2 +...< 1+ 2 = 3.


n


 n
n!   
 3  , từ đây ta suy ra
Vậy (*) đúng với n k  1 . Do đó

n

n! 

n
3

1
3
=> 0 < n ! < n .
n



lim

n 

3
n =0.

Do đó theo định lý về giới hạn kẹp giữa ta suy ra:
lim(2014 
Vậy


x 
Cho dãy số n
Tính I lim xn

n

lim

n 

1
n
n! = 0 .

1
)
n ! =2014

 x1 1; x2 2

5
xn 1 

2

; n  *
2
 xn 2  
4  xn 

thoả mãn 

Từ giả thiết suy ra mội số hạng của dãy đều dương
 y1 0; y2 1

2 y 5 yn 1  2 yn ; n  *
Đặt yn log 2 xn , ta có dãy  n 2
 z1  2; z2  1

2 z 5 zn 1  zn ; n  *
y

z

2
n
n
Lại đặt
, ta có dãy  n 2
1
zn  4. n
2 .
Tìm được số hạng tổng qt của dãy là
Từ đó ta có lim yn 2  lim xn 4 .

z 1 =−2 , z 2 =−1
2 z n+2 =5 z n+1 −z n

{


3.4. CÁC DẠNG KHÁC

Bài 12.

 x1  2016


m
*
 x n+1 1+x 2 n  N
n
Tìm các giá trị thực của tham số m để dãy số (xn): 
có giới hạn hữu hạn.
Hướng dẫn giải:

*) m > 0  0  xn  m n  1
Xét hàm số:

f ( x) 

 2mx
m
f '( x )  2

 0; m 
( x  1) 2
x 2  1 ta có
f(x) nghịch biến trên

Suy ra ( x2 n ), ( x2 n 1 ) đơn điệu và bị chặn.

0m

 x  x3  x5  ... 
2017
 x1  x2 , x3   1
2016
 x2  x4  x6  ... 

f ( f (1)) 

4m
m
1, x2 
 1  x2 n  1 n  N *
2
m 4
2017

+


Giả sử

2
a (1  b ) m
lim x2 n a, lim x2 n 1 b  a  1, 
(I )
2
b(1  a ) m


  a b
( II )
 3
a

a

m



( I )   b  1
a
 
( III )

1
  a  m
a
 
Khi o  m 2 hệ (I) có nghiệm duy nhất  (xn) có giới hạn hữu hạn.
2m
Khi

2017
2016 hệ (II) có nghiệm duy nhất lớn hơn 1 và hệ (III) có nghiệm thỏa mãn a b . Do đó

 lim x2 n lim x2 n 1  ( xn ) khơng có giới hạn.



2017
m  2017 2016  x1  x2 , x1  x3 
2016

 x1 x3  x5 ...

 x2  x4  x6 ...

 lim x2 n  lim x2 n 1  ( xn ) khơng có giới hạn.
*
+ m 2017 2016  xn  2016 n  N  l imxn  2016

 x  x3  x5  ...
m  2017 2016  x1  x2 , x1  x3   1
 x2  x4  x6  ...
+
 lim x2 n  lim x2 n 1  ( xn ) khơng có giới hạn.
*) m  0 tượng tự ta có 0  m 2 và m  2017 2016 .
x 
Bài 13. Cho số thực a, xét dãy số n n 1 được xác định bởi
xn3  6 xn  6
x1 a, xn 1  2
, n 1, 2,....
3 xn  9 xn  7
Tìm tất cả các giá trị của a để dãy số có giới hạn hữu hạn, tìm giới hạn đó?
Hướng dẫn giải:
lim xn  1
Với a  1 thì xn  1, n 1 nên n  
3
3

xn  1  1
xn  1  2 


xn  1  2
, xn  2  2
, n 2
3
x

9
x

7
3
x

9
x

7
a

1
n

1
n

1

n

1
n

1
Với
thì
3

Do đó

xn  2  xn  1  2 
 a2

 ... 

xn  1  xn  1  1 
 a 1 
xn 

2  a  1

3n 1

3n 1

3n 1

, n 1


  a  2

3n 1

3n 1

, n 1

 a  2    a  1
Từ đó, tính được
3
a    a  1  a  2  lim xn  2
n  
2
Kết luận +

,


3
 a  1  a  2  lim xn  1
n 
2
+
3
3
3
a   xn  , n 1  lim xn  .
n  

2
2
2
+
a

Bài 14.

Cho hai dãy số dương

 an  n0 ,  bn  n0

xác định bởi: a0  3, b0 2 và

1  an 1

 an  bn 1  a
n 1

 a 2  1 b 2
n
 n
Với mọi n 0,1, 2,... . Chứng minh rằng hai dãy trên hội tụ và tìm giới hạn của chúng.
Hướng dẫn giải:
an tan
Ta chứng minh bằng quy nạp


1
, bn 

, n 0,1, 2,... (*)
n

3.2
cos n
3.2
. Thật vậy



1
tan 0 , b0 2 

3
3.2
cos 0
3.2 , vậy  * đúng
Với n 0 , ta có
1


2
1
a1 
tan tan 1 , b1 

6
3.2
3
3 cos 

3.21 , vậy  * đúng
Với n 1 , ta có

1
an tan
, bn 
n

3.2
cos n
n

k
,
k

1
3.2
Giả sử khẳng định đúng đến
, tức là

1
an 1 tan
, bn 1 
n 1

3.2
cos n 1
3.2 . Thật vậy. Từ  1 ta có
Ta chứng minh






sin n  1 2sin n 1 cos n 1  sin 2
 cos 2
n

1
1  an 1
3.2
3.2
3.2
3.2n 1 
 3.2




1  an 1
cos n
cos 2
 sin 2
n 1
3.2
3.2
3.2n 1
a0  3 tan


2


 




sin n 1  cos n 1 tan n 1  1
 sin n 1  cos n 1 
3.2
3.2 

3.2 
3.2

 3.2
 a









 cos
 sin n 1 1  tan n 1
 cos n 1  sin n 1   cos n 1  sin n 1 

3.2n 1
3.2
3.2
3.2
3.2  
3.2
3.2 


1
1
bn21 an21  1 tan 2
1 
 bn 1 
n 1


3.2
cos 2
cos n 1
n 1
2

3.2
3.2
Khi đó từ
, suy ra

1
, bn 

, n 0,1, 2,...
n

3.2
cos n
3.2
Như vậy theo nguyên lý quy nạp thì

1
1
lim an  lim tan n tan 0 0; lim bn  lim

1
n  
n  
n  

3.2
cos
0
n  
cos n
3.2
Do đó
an tan

n 1

tan



3.2n 1


lim an 0; lim bn 1

Kết luận:

n  

Bài 15.

Cho dãy số (un ) xác định như sau :

n  

.■

u1 2014

2
2
un 1 un  (1  2a )un  a ; n 1, 2,...
Tìm điều kiện của a để dãy số (un ) có giới hạn hữu hạn khi n   và tính giới hạn đó.
Hướng dẫn giải
2
Ta có: un 1  un (un  a) 0  un 1 un ; n 1, 2,3,...
* Suy ra dãy số (un ) tăng knn ; từ đó dãy số (un ) có giới hạn hữu hạn khi và chỉ khi dãy bị chặn trên.
2
2

lim un L ( L  )
Giả sử n  
, thì chuyển qua giới hạn hệ thức un 1 un  (1  2a )un  a ta có:
L L2  (1  2a ) L  a 2  L a
lim un L a
*
- Nếu có chỉ số k   mà uk  a thì un  a; n k trái với kết quả n  
2
2
Do đó: uk a với mọi k 1, 2,... hay un  (1  2a)un  a a, n 1, 2,3,...
 a  1 u1 a  a  1 2014 a
* Đảo lại: Nếu a  1 2014 a  a  1 u1 a
 (u1  a  1)(u1  a ) 0  u12  (1  2a )u1  a 2  a 0  u2 a
và u1 u2  a  1 u2 a
Bằng quy nạp ta chứng minh được a  1 un a, n 1, 2,3,... (H/s trình bày ra)
Như vậy dãy (un ) tăng knn, bị chặn trên bới a , do đó dãy số (un ) có giới hạn hữu hạn.
lim un a
Kết luận: Với điều kiện a  1 2014 a thì dãy số (un ) có giới hạn hữu hạn khi n   và n  
.

Bài 16.

Cho dãy số

 xn 

 x1 a

2 xn3


 xn 1  3 x 2  1 , n 1, 2,3,...
n
thỏa mãn 

Tìm a sao cho dãy số xác định và có giới hạn hữu hạn.
Hướng dẫn giải
3

Đặt

f  x 

2x
3
, x 
2
3x  1
3 . Ta có x1 a, xn 1  f  xn  . Ta có
f ' x 

6x4  6x2

 3x

2

 1

2




6 x 2  x 2  1

 3x

2

 1

2

.

Bảng biến thiên
x


f’(x)


-1

3
3

0

3
3


1




-1







0
f(x)







1

Ta xây dựng dãy số như sau

a0 

3

, a0  f  a1  , a1  f  a2  , a2  f  a3  ,...
3
.

Nhận thấy a1 , a3 ,..., a2 k 1 ,...  0; a0 , a2 ,..., a2 k ,...  0 .


3 
3
a1   
;0  , a2  f  1  a1    0;

3 
3 


Dựa vào bảng biến thiên ta thấy
 a2  a0  f  a3   f  a1   a3  a1  f  a4   f  a2   a4  a2

Bằng quy nạp ta chứng minh được dãy

tăng và bị chặn bởi
Ta có



 a2k 

.


3
a 
đơn điệu giảm, bị chặn bởi 0 và 3 , dãy 2 k 1 đơn điệu

3
lim  a  , lim  a 
3 và 0. Từ đó tồn tại k   2 k k   2 k 1 .

an  f  an 1   f  f  an 2    lim an  f  f  lim an 2    l  f  f  l  
3

 2l 3 
2 2 
3l  1 
l 

2
 2l 3 
3 2   1
 3l  1 

1

l  l 2    l 2  1  20l 4  15l 2  5  0
5

(*)


3 

2 x3
3


f  x   2 , x 
 3 ;0 
 ,
3
x

1
3
(do
liên tục trên 

Xét

0l 


3
 0;

l  lim an
 3  và
n  
).

1
3

3
5
* 
 an 

l

3 . Vậy
5
3 . Ta có f  f  an    an an 2  an  0 nên
5 .

Tương tự ta chứng minh được dãy

 a2 k 1 

đơn điệu tăng, hội tụ về

 5

 5
xn 
5
 5
a
 5
5 thì x2  x1 , x3  x2 nên ta có dãy
+) Nếu
Dãy này khơng hội tụ.




5
5 .

nÕu n chẵn
nếu n lẻ


 5

 5
xn 
5
 5
a 
 5
5 ta có dãy
+) Nu

nếu n chẵn
nếu n lẻ

Dóy ny khụng hi t.
+) Nu tồn tại n sao cho a an thì ta có
x1 an  f  x1   f  an   x2 an  1  f  x2   f  an  1   x3 an  2 ,..., xn 1 a0 

3
3


Khi đó khơng tồn tại xn 2 .
Vậy nếu a an thì dãy khơng xác định.

+) Nếu

0a

5
5 thì hai dãy con  x2 k  ,  x2 k 1  cùng hội tụ về 0 nên giới hạn của dãy là 0.

x  f  a   a  x1
Nếu a  1 thì 2
và hàm số đồng biến nên dãy đơn điệu giảm, bị chặn dưới bởi 1. Khi đó
dãy hội tụ về 1.
3
 a 1
x  f  a 1
+) Nếu 3
thì 2
. Khi đó ta có thể khảo sát dãy từ x2 . Trường hợp này dãy đơn điệu
giảm và bị chặn dưới bởi 1 nên hội tụ về 1 .
+) Nếu a = 1 thì xn 1 n nên dãy hội tụ về 1 .

+) Nếu

5
3
5
3
a

 lim a2 n
a0 
n


5
3 ta có 5
3 nên tồn tại a2 k , a2 k 2 sao cho a2 k 2  a  a2 k (Thật


vậy, các số hạng của

 a2k 

bên phải a do nếu thế thì

Vậy

khơng thể cùng nằm bên trái a do

a  a2 n 

a0 

3
3 , chúng cũng không thể cùng nằm

3
5
 lim a2 n 

n  
3
5 ).

a   a2 k 2 ; a2 k   x2   a2 k ; a2 k  2  ,..., x2 k   a2 ; a0  , x2 k 2   a0 ;    x2 k 2 

3
3 . Khi đó ta lại có

dãy đơn điệu giảm, bị chặn dưới bởi 1 nên hội tụ về 1.

Vì f(x) là hàm lẻ nên trường hợp



5
3
 a  0,  1  a  
, a   1, a  1
5
3
ta khảo sát tương tự.

Kết luận: Điều kiện để dãy xác định và có giới hạn hữu hạn là
a 

3
5
; a  ; a an , n 1, 2,3,...
3

5


Bài 17.

Cho dãy số

Chứng minh rằng

 an 

xác định bởi 0  a1 1 và

lim  an  n  0
n 

an 1 an 

n
, n 1
an
.

.

Hướng dẫn giải
1
a2 a1   2
a1
Áp dụng bất đẳng thức AM-GM ta có

(do a1 1 )
Nhận xét: an  n, n 2 .
Ta sẽ chứng minh nhận xét này bằng phương pháp quy nap.
Thật vậy


Với n 2 ta có a2  2 (đúng)



Giả sử ak  k



Ta có

ak 1 ak 

k
 k  1  ak2  k   k  1 ak
ak
 ak2   k  1 ak  k  0

  ak  1  ak  k   0

(đúng)

Suy ra ak 1  k  1
Như vậy an  n, n 2 (điều phải chứng minh).
n

n
an 1   n  1 an    n  1 an  n   1
an
an
Mặt khác,


an2   n  1 an  n  an  n   an  1

an
an

(1)

Áp dụng (1) ta có

 a2  2   a2  1
 a3  3 
a2


 a  3  a3  1
 a4  4  3
a3


...


 an  n   an  1

 an 1   n  1 
an


Suy ra

 a3  3  a4  4  ...  an1   n  1  

 an 1   n  1 

 a2  2   a2  1  a3  3  a3  1 ...  an  n   an  1
a2 a3 ...an

 a2  2   a2  1  a3  1 ...  an  1
a2 a3 ...an

.



1 
1 
1
 an 1   n  1  a2  2   1 
 ...  1 

 1
 a2   a3   an 
n


1
 an 1   n  1  a2  2   1  
ai 
i 2 
(2)
n
an   1
an 1  1
an
a
1
n
1


 n
an  n 
1
an 1
an 1
an 1
an 1 (do
an
Ta lại có
)
n

Suy ra




 1 
i 2



1
ai

 a1 a2 an  1 a1

  . ...
an
 a2 a3 an
.

 an 1   n  1   a2  2  .

Từ (2)

a1
a
  a2  2  . 1
an
n (vì an  n )

 0  an 1   n  1   a2  2  .

a1
n.


a1
a
0  lim  a2  2  1 0
n 
n
Mà n   n
.
lim

Do đó

lim  an 1   n  1  0
n 

Bài 18.

hay

lim  an  n  0
n 

.

*
Cho p   , a  0 và a1  0 . Xét dãy số ( an ) được xác định bởi:

1
a 
an 1   ( p  1)an  p  1 

p
an 
, với mọi n 1 .
Chứng minh dãy số ( an ) có giới hạn hữu hạn khi n  . Hãy tìm giới hạn đó.
Hướng dẫn giải
* Theo bất đẳng thức Cơsi ta có:
1 
a 
1
a
p
an 1  an  an  ...  an  p  1  p. p anp  1. p  1  a







p
an 
p
an
p 1


, với  n 1 . (1)
1
a 
an 1  an   ( p  1)an  p  1   an

p
an 
a
anp  a
a
 n 

0;  n 2
p p.anp  1
p.anp  1

Do đó:

Từ (1) và (2) ta có dãy số ( an ) giảm và bị chặn dưới bởi
suy ra dãy số ( an ) có giới hạn hữu hạn khi n  .
Giả sử

lim an L

n  

p

a;

p

; ( L  a ).

1

a 
an 1   ( p  1)an  p  1 
p
an 
Chuyển qua giới hạn hệ thức
1
a 
L   ( p  1) L  p  1   pLp ( p  1) Lp  a
p
L 
ta có phương trình

(2)


p

 Lp a  L  a (thỏa mãn điều kiện).
p
lim an  a
Vậy n  
.

x 
Cho trước số thực dương  và xét dãy số dương n thỏa mãn

Bài 19.

xn1 
Chứng minh rằng dãy


 xn 



1
    1   1
*
xn
với mọi n   .

hội tụ và tìm giới hạn của nó.

Hướng dẫn giải
1
f ( x )  x  , x  0
x
Xét hàm số
.
 1
1 x  1
1

f '( x )  x  1  2 
 1
2
f
'(
x
)


0

x

x


0
x
x
Ta có
;
.
Ta có bảng biến thiên của hàm f(x):
x

x0

0

0

f'(x)

+∞
+

+∞


+∞

f(x)
f(x0)

f ( x )  f  x0  

Suy ra
Do đó

xn1 




 1



1
 1

(  1)




 1




1
1
    1   1  xn1 
xn
xn 1

x 
x 
Suy ra xn 1  xn hay n là dãy giảm. Kết hợp với xn  0 với mọi n ta suy ra dãy n hội tụ.


1

 1
  (  1)
   x0

Đặt lim xn   0 . Chuyển qua giới hạn ta được
Vậy lim xn 
Bài 20.



1
 1

.

Tìm tất cả các hằng số c  0 sao cho mọi dãy số dãy số (un ) thỏa mãn

un  (0;1)
n 1

un 1 (1  un )  c

đều hội tụ. Với giá trị c tìm được hãy tính giới hạn của dãy (un ) .
Hướng dẫn giải
Ta xét các trường hợp sau
+ Nếu

c

cun
c
1
un 1 

4cun ; n 1
1  un un (1  un )
4 , thì từ giả thiết, ta có


1
c
n 1
u

(4
c
)

u
u


1 . Do 4c  1 nên
n
4
Từ đây bằng quy nạp, ta suy ra n
khi n   . Do đó,
khơng thỏa mãn
 1  1  4c 1  1  4 c 
 a (1  b)  c
1
a
,
b

;

 , a  b

0c
2
2


4 , thì tồn tại
+ Nếu
sao cho b(1  a )  c . Thật vây, lấy
 1  1  4c 1  1  4 c 

a  
;
 ,
2
2

 đặt b a  x ( x  0) , thì
a(1  a )  c
a (1  b)  c  a (1  a  x )  c  x 
a
.
Chú ý là b(1  a)  a(1  a)  c. Do đó, ta chỉ cần chọn x  0 như trên và b a  x, thì được 2 bất đẳng
thức nêu trên.
Xét dãy số (un ) xác định bởi
a khi n 2m
un 
b khi n 2m  1
1
0c
(
u
)
4 cũng khơng thỏa mãn.
thì dãy n thỏa mãn giả thiết nhưng không hội tụ. Thành thử,
un
1
1
un 1 

un

c
4(1

u
)
4
u
(1

u
)
4
n
n
n
+ Nếu
, thì
. Suy ra dãy (un ) tăng và bị chặn. Do đó, (un ) hội tụ.
1
1
1
x(1  x) 
x .
lim un  .
x

li
m
u
,

n thì từ giả thiết ta có
4 hay
2 Vậy
2
Đặt
Bài 21.

Cho dãy số

 sn 

với

 un 

sn 

2
*
xác định như sau: u1 2 , un 1 un  un  1 , n   . Tìm giới hạn của dãy

1 1
1
  ...  , n  *
u1 u2
un
.
Hướng dẫn giải

Bằng phép quy nạp đơn giản ta thấy rằng: un 2 .

2
u 
Xét tính đơn điệu của dãy n . Từ hệ thức un 1 un  un  1 ta suy ra được
2
n  * , un 1  un  un  1  0
u 
, vậy dãy số n tăng.
Tính tổng: Từ hệ thức truy hồi (1) ta suy ra được


1
un 1  1



un 1  1 un  un  1

1
1
1
1
1
1





un  un  1 un  1 un
un un  1 un 1  1


 *

*
với n  

Thay n bởi 1, 2, 3, ... , n vào (*) và cộng vế với vế các đẳng thức ta suy ra :
1 1
1
1
  ...  1 
u1 u2
un
un 1  1
Do dãy

 un 

là dãy tăng nên có hai khả năng sau xảy ra:


u 
bị chặn trên. Theo tiêu chuẩn Weierstrass, nên n tăng và bị chặn trên nên nó có giới hạn.
lim un a  a 2
Giả sử n  
. Chuyển qua giới hạn hệ thức (1) khi n   ta có:
2
2
a a  a  1  a  2a  1 0  a 1 , vô lý.
1) Dãy


 un 

2) Dãy không bị chặn trên, do

 un 

tăng và không bị chặn trên nên
1
lim un   lim  un  1   lim
0
n  
n  
n   u
n
1 1

1
1
lim    ...    lim  1 
 1
n   u
u2
un  n   un 
1

Vì thế từ (2) ta suy ra:

Bài 22.


Cho dãy số (un) thỏa mãn :

u0 2016; un 1 un 

1
un2 .

un3
Tính n   n .
lim

Hướng dẫn giải
3


1 
3 1
(un 1 )3  un  2  un3  3  3  6
un 
un un

3 1
(un 1 )3 un3  3  3  6  un3  3 , n
un un
Do un  0 n =>
3
3
3
suy ra (un )  u0  3n 2016  3n, n  
Lại có


(1)

3


1 
3 1
(un 1 )  un  2  un3  3  3  6
un 
un u n

3
1
1
1
 un3  3 

 un3  3  
2
3
3
2016  3n  2016  3n 
n  3n  2
3

1
1
(un 1 )3  un3  3  
n  

n  3n  2

=>
Suy ra

n 1

n
1 n 1 1
1 n 1
  2  u13  3n     2
k 1 k
k 1 9k
k 1 k
k 1 9k
n
1
1
1
1
1
1

 ... 
2   2

2
1.2 2.3
(n  1)n
n

k 1 k

(un )3  u13  3( n  1)  

Do

2

n
1
 n 1 

n
 2n


 k 
2
k 1 k
 k 1 

(Bất đẳng thức Bunhiacopxki)
2
(un )3  u13  3n   2n
9
suy ra
(2)
Từ (1) và (2) suy ra
2
20163  3n  (un )3  u13  3n   2n , n  

9
3
3
3
(u )
u
2016
2
2

3  n  1 3 
, n  
n
n
n
9n
n


un3
3
Do đó n   n
lim

Bài 23.

Cho số thực a, xét dãy số

 xn  xác định bởi:


x1 a, xn 1 ln  3  cos xn  sin xn   2014, n 1, 2...
Chứng minh rằng dãy số trên có giới hạn hữu hạn khi n  .
Hướng dẫn giải
Đặt

f  x  ln  3  sin x  cos x   2014, x  

cos x  sin x
3  sin x  cos x


 3 f '  x   2 cos  x   
4

 f ' x 

2



2 f '  x  sin  x  
4


2

 9  f '  x   2  2  f '  x    f '  x  
Áp dụng định lí Lagrange cho hàm số

f  x


2
q, x  
7
.
liên tục và có đạo hàm trên  , thì với mọi số thực x,y tồn tại

z   sao cho:
f  x   f  y   f '  z  x  y q x  y  f  x   f  y  q x  y , x , y  
m  n  m, n  *  ,

x  x  f  xm  1   f  xn  1  q xm  1  xn  1 ... q m n  1 xm  n 1  x1
ta có: m n
2014  xn  2014  ln 5, n  *   xn 
Mặt khác:
bị chặn
  0, N  * : q m n 1 xm n 1  x1   , m  n  N .
Do đó:
x 
Vậy n là dãy Cauchy, nên dãy số đã cho hội tụ.
Với

4. GIỚI HẠN CỦA HÀM SỐ
4.1. TÍNH GIỚI HẠN BẰNG ĐỊNH NGHĨA
4.2. TÍNH GIỚI HẠN BẰNG CÁC CƠNG THỨC CƠ BẢN
4.3. TÍNH GIỚI HẠN BẰNG ĐỊNH LÍ KẸP
4.3. TÍNH GIỚI HẠN BẰNG ĐẠO HÀM
4.4. CÁC DẠNG KHÁC




Tài liệu bạn tìm kiếm đã sẵn sàng tải về

Tải bản đầy đủ ngay
×